$\require{enclose}$ $\newcommand{\avsum}{\mathrel{\displaystyle\int \!\!\!\!\!\! \Delta\ }}$ $\newcommand{\bcancelto}[2]{{\enclose{southeastarrow}{#2}\,}_{\lower.75ex{#1}}}$ $\newcommand{\ordcirc}[1]{\mathrel{[\hspace{-4pt} \circ \hspace{2pt}#1 \hspace{3pt}]\hspace{-4pt}\circ}}$ $\newcommand{\avigual}{\{=\}}$ $\newcommand{\intsup}{{\LARGE \big\uparrow}\displaystyle\int}$ $\newcommand{\intinf}{{\LARGE \big\downarrow}\displaystyle\int}$
Última atualização estrutural do weblog: 07-07-2023.

Este weblog utiliza serviços de terceiros, e os mesmos podem não funcionar adequadamente, o que não depende de mim.

Se as expressões matemáticas não estiverem satisfatoriamente visíveis, você pode alterar as configurações de exibição no menu contextual.

Este weblog pode passar por melhorias. Caso não teve uma boa experiência hoje, futuramente os problemas poderão estar corrigidos.

Em caso de não ser a mim mais possível realizar manutenções, como, por exemplo, devido a falecimento ou desaparecimento, alguns links podem ficar quebrados e eu não responder mais a comentários. Peço compreensão.

quarta-feira, 5 de junho de 2013

Exercício: movimento oscilatório de um corpo flutuando em um fluido.

Uma rolha de densidade $d_r$ e altura $H$ flutua num líquido de densidade $d_{\ell}$. Afunda-se ligeiramente a rolha para baixo, deixando-se, a seguir, oscilar. Sendo $g$ a aceleração local da gravidade, determine o período de oscilação da rolha.

Resolução:

Antes de tudo, vamos determinar qual será o tipo de movimento da rolha.

Quando estática, o sistema estará em equilíbrio. Após afundar a rolha, surgirá uma resultante-empuxo trará a rolha de volta à sua posição inicial.

Chamando $A$ a área da secção reta da rolha, $E$ a força-empuxo, $h$ o comprimento submerso da rolha em equilíbrio, e $x$ o deslocamento imposto à rolha, teremos:

$E - P\ =\ d_{\ell}\ \cdot\ A\ \cdot\ (h - x)\ \cdot\ g\ -\ d_r\ \cdot\ A\ \cdot\ H\ \cdot\ g\ =$

$=\ gA[d_{\ell}(h - x) - d_r H]$

Considerando $d_r\ \approx\ 0$, $h\ \approx\ 0$, $P\ \approx\ 0$, e chamando de $R$ a resultante, teremos:

$R\ =\ E\ =\ -d_{\ell}gA\ \cdot\ x$

Logo podemos considerar $R$ aproximadamente uma força restauradora e diretamente proporcional ao deslocamento imposto $x$, caracterizando um MHS linear.

Assim podemos utilizar a fórmula geral do período:

$T\ =\ 2\pi\sqrt{\dfrac{m}{k}}$

Onde:

$m\ =\ d_r A H$

e

$k\ =\ d_{\ell}gA$

Logo:

$T\ =\ 2\pi\sqrt{\dfrac{d_r H}{d_{\ell} g}}$

terça-feira, 4 de junho de 2013

Exercício: período de oscilação de um pêndulo na Lua.

Na Terra, um pêndulo simples executa oscilações com período $T_T$. Se este pêndulo oscilasse na Lua, seu período seria $T_L$. Determine a razão $\dfrac{T_T}{T_L}$. Sabe-se que a aceleração da gravidade na Lua é seis vezes menor que na Terra.

Resolução:

Para pequenas oscilações, o movimento do pêndulo será aproximadamente harmônico simples linear, o que possibilitará-nos utilizar a fórmula $T = 2\pi\sqrt{\dfrac{\ell}{g}}$.

Como a aceleração da gravidade na Lua é seis vezes menor que na Terra, teremos: $g_L = \dfrac{g_T}{6}$.

Assim:

$T_L\ =\ 2\pi\sqrt{\dfrac{\ell}{\dfrac{g_T}{6}}}\ \Rightarrow\ T_L\ =\ (\sqrt{6})\ \cdot\ (2\pi\sqrt{\dfrac{\ell}{g_T}})\ =\ \sqrt{6}\ \cdot\ T_T$

Donde:

$\dfrac{T_T}{T_L}\ =\ \dfrac{T_T}{\sqrt{6}\ \cdot\ T_T}\ =\ \dfrac{1}{\sqrt{6}}$

domingo, 19 de maio de 2013

Exercício: lentes ópticas.

Usando uma lente delgada convergente de distância focal $f$ é possível projetar nitidamente a imagem de um objeto frontal sobre uma tela situada a uma distância $D$ do objeto. Verifica-se também que, dependendo da relação entre $f$ e $D$, há duas posições da lente que dão imagem nítida; às vezes uma só posição e, às vezes, nenhuma. Determine uma relação entre $f$ e $D$ para que haja formação de tal imagem nítida.

Resolução:

Como trata-se de uma lente delgada, podemos usar as relações de Gauss. Como projeta uma imagem em um anteparo, a imagem é real, e como temos uma lente convergente, o objeto será também real.

$D\ =\ p\ +\ p'\ \Rightarrow\ p'\ =\ D\ -\ p$

$\dfrac{1}{f}\ =\ \dfrac{1}{p}\ +\ \dfrac{1}{p'}\ =\ \dfrac{1}{p}\ +\ \dfrac{1}{D - p}$

$p^2\ -\ Dp\ +\ Df\ =\ 0$

Para determinar quantas imagens nítidas serão formadas, basta analisar quantas soluções terá a equação polinomial do segundo grau em $p$.

$\Delta\ =\ D^2 - 4Df$

Para termos duas imagens nítidas, teremos que $D\ >\ 4f$. Para termos uma única imagem nítida, teremos que $D\ =\ 4f$. Para não termos imagens nítidas, teremos que $D\ <\ 4f$.

quinta-feira, 4 de abril de 2013

Exercício: produto de matrizes #2.

Sendo A, B e C matrizes, tais que $C\ =\ A\ \cdot\ B$, com:

$A\ =\ (a_{i j})_{2 \times 3},\ a_{i j}\ =\ i^2 + j^2$;
$B\ =\ (b_{i j})_{3 \times 4},\ b_{i j}\ =\ 2i + j$;
$C\ =\ (c_{i j})$.

Determine os elementos $c_{2 3} $ e $ c_{3 4}$ da matriz $C$.

Resolução :

Por definição, o elemento $c_{2 3}$ será $P\ =\ \sum_{k = 1}^3 (a_{2 k}\ \cdot\ b_{k 3})$.

$P\ =\ \sum_{k = 1}^3 [(2^2 + k^2)\ \cdot\ (2k + 3)]\ =\ 5\ \cdot\ 5\ +\ 8\ \cdot\ 7\ +\ 13\ \cdot\ 9$

$P\ =\ 25\ +\ 56\ +\ 117\ =\ 198$

segunda-feira, 18 de fevereiro de 2013

Exercício: atraso na dilatação térmica de um relógio de pêndulo.

(Unisa-SP) Um relógio é controlado por um pêndulo que marca corretamente os segundos a $20 ^\circ C$. O pêndulo é feito de um material cujo coeficiente de dilatação linear é $16\ \cdot\ 10^{-6}\ ^\circ C^{-1}$. Quando a temperatura é mantida a $30\ ^\circ C$, qual o atraso do relógio em uma semana?

Resolução :

O período de um pêndulo que oscila em ângulos pequenos é dado aproximadamente por $T\ =\ 2\pi \sqrt{\dfrac{L}{g}}$, onde $L$ é seu comprimento e $g$ é a aceleração da gravidade local.

Sabemos também que, do teorema da dilatação térmica, $L\ =\ L_0 (1 + \alpha \Delta \theta)$.

Para os dados do enunciado, teremos $L\ =\ L (1 + 16\ \cdot\ 10^{-6}\ \cdot\ 10)\ =\ L\ \cdot\ 1,00016$.

Assim, se o comprimento será multiplicado por $1,00016$, o período do pêndulo será multiplicado por $\sqrt{1,00016}\ \approx\ 1,000080$.

Como em uma semana temos $7\ \cdot\ 24\ \cdot\ 3600\ =\ 604800$ segundos, o relógio com o pêndulo dilatado marcará $\dfrac{604800}{1,000080}\ \approx\ 604752$ segundos, dando uma diferença, considerando as aproximações, de $48$ segundos.

quinta-feira, 14 de fevereiro de 2013

Exercício: número de vasos capilares.

Em um ser humano adulto, a artéria aorta tem raio interno aproximadamente igual a $1,0\ cm$, e o sangue passa por ela com velocidade média igual a $30\ \dfrac{cm}{s}$. Em um vaso capilar o raio interno é aproximadamente igual a $4,0\ \cdot\ 10^{-4}\ cm$, e a velocidade do sangue é aproximadamente igual a $5,0\ \cdot\ 10^{-2}\ \dfrac{cm}{s}$. Calcule a ordem de grandeza do número de vasos capilares.

Resolução :

A vazão na aorta será :

$\Phi_a\ =\ A_a\ \cdot\ v_a\ =\ (1,0)^2\ \cdot\ \pi\ \cdot\ 30\ =\ 30 \pi\ \dfrac{cm^3}{s}$

A vazão em um capilar será :

$\Phi_c\ =\ A_c\ \cdot\ v_c\ =\ (4,0\ \cdot\ 10^{-4})^2\ \cdot\ \pi\ \cdot\ 5,0\ \cdot\ 10^{-2}\ =$

$=\ 8 \pi\ \cdot\ 10^{-9}\ \dfrac{cm^3}{s}$

Chamemos de $n$ o número de capilares. Como a vazão da aorta se distribui para todos os capilares, teremos :

$\Phi_a\ =\ n\ \cdot\ \Phi_c$

$30 \pi\ =\ n\ \cdot\ 8 \pi \cdot\ 10^{-9}$

$n\ =\ 3,75\ \cdot\ 10^9$

Logo a ordem é de bilhões de vasos capilares em um humano adulto.

segunda-feira, 31 de dezembro de 2012

Demonstração: $f$ crescente se $f^{-1}$ também crescente.

Seja $f$ uma função real de variável real e inversível.

Se $f$ é crescente, teremos :

Sejam $x_2 , x_1\ \in\ Dom(f)$:

$x_2 > x_1\ \Leftrightarrow\ f(x_2) > f(x_1)$

$f^{-1}[f(x_2)] > f^{-1}[f(x_1)]\ \Leftrightarrow\ f(x_2) > f(x_1)$

Assim, se $f$ é crescente, $f^{-1}$ também o é, e, reciprocramente, se $f$ é decrescente, $f^{-1}$ também o é.